Diễn Đàn MathScopeDiễn Đàn MathScope
  Diễn Đàn MathScope
Ghi Danh Hỏi/Ðáp Thành Viên Social Groups Lịch Ðánh Dấu Ðã Ðọc

Go Back   Diễn Đàn MathScope > Sơ Cấp > Đại Số và Lượng Giác

News & Announcements

Ngoài một số quy định đã được nêu trong phần Quy định của Ghi Danh , mọi người tranh thủ bỏ ra 5 phút để đọc thêm một số Quy định sau để khỏi bị treo nick ở MathScope nhé !

* Nội quy MathScope.Org

* Một số quy định chung !

* Quy định về việc viết bài trong diễn đàn MathScope

* Nếu bạn muốn gia nhập đội ngũ BQT thì vui lòng tham gia tại đây

* Những câu hỏi thường gặp

* Về việc viết bài trong Box Đại học và Sau đại học


Trả lời Gởi Ðề Tài Mới
 
Ðiều Chỉnh Xếp Bài
Old 08-04-2011, 09:54 PM   #181
vthiep94
+Thành Viên+
 
Tham gia ngày: Dec 2009
Bài gởi: 197
Thanks: 185
Thanked 49 Times in 31 Posts
Trích:
Nguyên văn bởi hoaquynh1995 View Post
Bài 68:
Cho $a, b, c $ là các số không âm. Chứng minh rằng $a^2\left( \frac{b}{c}-1 \right)+b^2\left( \frac{c}{a}-1 \right)+c^2\left( \frac{a}{b}-1 \right) \ge 0 $
bài này thiếu điều kiện !!!
------------------------------
Bài 69: cho a,b,c dương và abc=1. CM
$\[
\sum {\frac{{a + b + 1}}{{a + b^2 + c^3 }} \le \frac{{(a + 1)(b + 1)(c + 1) + 1}}{{a + b + c}}}
\]
$
Bài 70 : Cho a,b,c dương vài $a^2+b^2+c^2=3 $
Cm
$\[
\sum {} \frac{1}{{(2 - ab)(2 - ac)}} \le 3
\]
$
p/s : Bài 70 là bài mình tự chế, mọi người giải thử xem sao
[RIGHT][I][B]Nguồn: MathScope.ORG[/B][/I][/RIGHT]
 

thay đổi nội dung bởi: vthiep94, 08-04-2011 lúc 10:29 PM Lý do: Tự động gộp bài
vthiep94 is offline   Trả Lời Với Trích Dẫn
Old 08-04-2011, 11:26 PM   #182
batigoal
Super Moderator
 
batigoal's Avatar
 
Tham gia ngày: Jul 2010
Đến từ: Hà Nội
Bài gởi: 2,895
Thanks: 382
Thanked 2,968 Times in 1,295 Posts
Trích:
Nguyên văn bởi vthiep94 View Post
Bài 69: cho a,b,c dương và abc=1. CM
$\[
\sum {\frac{{a + b + 1}}{{a + b^2 + c^3 }} \le \frac{{(a + 1)(b + 1)(c + 1) + 1}}{{a + b + c}}}
\]
$
Ta có:$\sum {\frac{{a + b + 1}}{{a + b^2 + c^3 }} \le\sum \frac{(a+1+ab)(a+b+1)}{(a+b+c)^2}=(a+b+c) \frac{\sum ab+\sum a+3}{(a+b+c)^2} $
$=\frac{\sum ab+\sum a+3}{a+b+c}=\frac{{(a + 1)(b + 1)(c + 1) + 1}}{{a + b + c}}} $
[RIGHT][I][B]Nguồn: MathScope.ORG[/B][/I][/RIGHT]
 
__________________
“ Sức mạnh của tri thức là sự chia sẻ tri thức”

[Only registered and activated users can see links. ]
batigoal is offline   Trả Lời Với Trích Dẫn
The Following 3 Users Say Thank You to batigoal For This Useful Post:
manhnguyen94 (09-05-2011), mrvui123 (09-09-2012), vthiep94 (16-04-2011)
Old 11-04-2011, 11:48 AM   #183
Nguyenhuyen_AG
+Thành Viên+
 
Nguyenhuyen_AG's Avatar
 
Tham gia ngày: Apr 2010
Bài gởi: 300
Thanks: 35
Thanked 307 Times in 151 Posts
My problem

Bài 71. Cho $a,b,c $ là các số thực không âm sao cho $a+b+c>0. $ Chứng minh rằng
$\frac{a+2b}{2a+4b+3c}+\frac{b+2c}{2b+4c+3a}+\frac{ c+2a}{2c+4a+3b}\le 1 $

[RIGHT][I][B]Nguồn: MathScope.ORG[/B][/I][/RIGHT]
 
__________________
Nguyen Van Huyen
Ho Chi Minh City University of Transport

thay đổi nội dung bởi: phantiendat_hv, 11-04-2011 lúc 11:56 AM
Nguyenhuyen_AG is offline   Trả Lời Với Trích Dẫn
Old 11-04-2011, 12:36 PM   #184
buon qua
+Thành Viên+
 
buon qua's Avatar
 
Tham gia ngày: Jan 2010
Bài gởi: 39
Thanks: 70
Thanked 56 Times in 23 Posts
Trích:
Nguyên văn bởi Nguyenhuyen_AG View Post
Bài 71. Cho $a,b,c $ là các số thực không âm sao cho $a+b+c>0. $ Chứng minh rằng
$\frac{a+2b}{2a+4b+3c}+\frac{b+2c}{2b+4c+3a}+\frac{ c+2a}{2c+4a+3b}\le 1 $
Bất đẳng thức cần chứng minh tương đương với:
$\frac{c}{2a+4b+3c}+\frac{a}{2b+4c+3a}+\frac{b}{2c+ 4a+3b} \ge \frac{1}{3}. $
Sử dụng Cauchy-Schwarz ta có:
$P=\sum\frac{c}{2a+4b+3c}=\sum\frac{c^2}{2ac+4bc+3c ^2} \ge \frac{(a+b+c)^2}{3(a+b+c)^2}=\frac{1}{3}. $

[RIGHT][I][B]Nguồn: MathScope.ORG[/B][/I][/RIGHT]
 
buon qua is offline   Trả Lời Với Trích Dẫn
The Following User Says Thank You to buon qua For This Useful Post:
daylight (16-04-2011)
Old 14-04-2011, 11:25 PM   #185
Persian
+Thành Viên+
 
Tham gia ngày: Nov 2010
Đến từ: Có những thứ mình đã nhẫn tâm đánh mất sẽ không bao giờ lấy lại được.
Bài gởi: 257
Thanks: 103
Thanked 200 Times in 112 Posts
Tiếp mọi người
Bài 72:Chứng minh rằng nếu $a,b,c $ là các số dương thì
$\frac{abc}{(a+b)(b+c)(c+a)} \le \frac{(a+b)(a+b+2c)}{(3a+3b+2c)^2} \le \frac{1}{8} $
{Trần Quốc Luật}
[RIGHT][I][B]Nguồn: MathScope.ORG[/B][/I][/RIGHT]
 
Persian is offline   Trả Lời Với Trích Dẫn
Old 15-04-2011, 08:39 PM   #186
supermouse
+Thành Viên+
 
supermouse's Avatar
 
Tham gia ngày: Mar 2010
Đến từ: diamond planet
Bài gởi: 85
Thanks: 10
Thanked 45 Times in 29 Posts
Gửi tin nhắn qua Yahoo chát tới supermouse
Trích:
Nguyên văn bởi Persian View Post
Tiếp mọi người
Bài 72:Chứng minh rằng nếu $a,b,c $ là các số dương thì
$\frac{abc}{(a+b)(b+c)(c+a)} \le \frac{(a+b)(a+b+2c)}{(3a+3b+2c)^2} \le \frac{1}{8} $
{Trần Quốc Luật}
BĐT (1)
$\begin{array}{l}
\Leftrightarrow \frac{{(a + b)(a + c)(b + c)}}{{abc}} \ge \frac{{{{(3a + 3b + 2c)}^2}}}{{(a + b)(a + b + 2c)}}(*) \\
VT + 1 = \frac{{(a + b + c)(ab + bc + ac)}}{{abc}} = (a + b + c)(\frac{1}{a} + \frac{1}{b} + \frac{1}{c}) \\
a + b = x;a + b + 2c = y \\
\Rightarrow VP + 1 = \frac{{{{(2x + y)}^2} + xy}}{{xy}} = \frac{{4{x^2} + 5xy + {y^2}}}{{xy}} = \frac{{(4x + y)(x + y)}}{{xy}} = (a + b + c)(\frac{8}{{a + b + 2c}} + \frac{2}{{a + b}}) \\
\Rightarrow (*) \Leftrightarrow \frac{1}{a} + \frac{1}{b} + \frac{1}{c} \ge \frac{8}{{a + b + 2c}} + \frac{2}{{a + b}} \\
\end{array} $


[RIGHT][I][B]Nguồn: MathScope.ORG[/B][/I][/RIGHT]
 
__________________
NEVER GIVE UP☺☺☺☺☺☺☺☺
supermouse is offline   Trả Lời Với Trích Dẫn
The Following 4 Users Say Thank You to supermouse For This Useful Post:
daylight (16-04-2011), duynhan (15-04-2011), Persian (17-04-2011), vthiep94 (18-04-2011)
Old 17-04-2011, 07:12 PM   #187
batigoal
Super Moderator
 
batigoal's Avatar
 
Tham gia ngày: Jul 2010
Đến từ: Hà Nội
Bài gởi: 2,895
Thanks: 382
Thanked 2,968 Times in 1,295 Posts
Bài 73 (Vasc)
Cho ba số thực $a,b,c $ không âm và hai trong ba số không đồng thời bằng 0.Chứng minh rằng:

$\frac{1}{4a^2+b^2+c^2}+\frac{1}{4b^2+c^2+a^2}+ \frac{1}{4c^2+a^2+b^2}\leq \frac{1}{2(a^2+b^2+c^2)}+\frac{1}{ab+bc+ca} $

[RIGHT][I][B]Nguồn: MathScope.ORG[/B][/I][/RIGHT]
 
__________________
“ Sức mạnh của tri thức là sự chia sẻ tri thức”

[Only registered and activated users can see links. ]
batigoal is offline   Trả Lời Với Trích Dẫn
Old 17-04-2011, 09:33 PM   #188
Thanh vien
+Thành Viên+
 
Thanh vien's Avatar
 
Tham gia ngày: Oct 2009
Bài gởi: 120
Thanks: 68
Thanked 70 Times in 40 Posts
Icon1

Trích:
Nguyên văn bởi batigoal View Post
Bài 73 (Vasc)
Cho ba số thực $a,b,c $ không âm và hai trong ba số không đồng thời bằng 0.Chứng minh rằng:

$\frac{1}{4a^2+b^2+c^2}+\frac{1}{4b^2+c^2+a^2}+ \frac{1}{4c^2+a^2+b^2}\leq \frac{1}{2(a^2+b^2+c^2)}+\frac{1}{ab+bc+ca} $
VP = $\frac12\left(\frac1{a^2+b^2+c^2}+\frac2{ab+bc+ca} \right)\ge\frac12.\frac9{(a+b+c)^2} $
Cần chứng minh
$\frac9{2(a+b+c)^2}\ge\sum\frac1{4a^2+b^2+c^2} $
hay $\sum\frac{(a+b+c)^2}{4a^2+b^2+c^2}\le\frac92 $
Đúng theo Schwarz vì
$\sum\frac{(a+b+c)^2}{4a^2+b^2+c^2}\le\sum\left( \frac{a^2}{2a^2}+\frac{b^2}{a^2+b^2}+\frac{c^2}{a^ 2+c^2}\right)=\frac92 $
Ta có đpcm.
[RIGHT][I][B]Nguồn: MathScope.ORG[/B][/I][/RIGHT]
 
Thanh vien is offline   Trả Lời Với Trích Dẫn
The Following 2 Users Say Thank You to Thanh vien For This Useful Post:
daylight (18-04-2011), mrvui123 (16-09-2012)
Old 17-04-2011, 10:34 PM   #189
batigoal
Super Moderator
 
batigoal's Avatar
 
Tham gia ngày: Jul 2010
Đến từ: Hà Nội
Bài gởi: 2,895
Thanks: 382
Thanked 2,968 Times in 1,295 Posts
Trích:
Nguyên văn bởi Thanh vien View Post
VP = $\frac12\left(\frac1{a^2+b^2+c^2}+\frac2{ab+bc+ca} \right)\ge\frac12.\frac9{(a+b+c)^2} $
Cần chứng minh
$\frac9{2(a+b+c)^2}\ge\sum\frac1{4a^2+b^2+c^2} $
hay $\sum\frac{(a+b+c)^2}{4a^2+b^2+c^2}\le\frac92 $
Đúng theo Schwarz vì
$\sum\frac{(a+b+c)^2}{4a^2+b^2+c^2}\le\sum\left( \frac{a^2}{2a^2}+\frac{b^2}{a^2+b^2}+\frac{c^2}{a^ 2+c^2}\right)=\frac92 $
Ta có đpcm.
Bất đẳng thức của Vasc rất đẹp và cách chứng minh của ThanhVien đẹp cũng không chê vào đâu được.
Mình xin phép được trình bày ý tưởng và cách tiếp cận của ThanhVien khi giải quyết bài toán này:

Quan sát BDT chúng ta thấy vế phải của BDT mẫu số có $a^2+b^2+c^2 $ và $ab+bc+ca $. Chúng có mối liên hệ với nhau là hằng đẳng thức $(a+b+c)^2=a^2+b^2+c^2+2(ab+bc+ca) $ nên bạn ThanhVien xuấ phát từ VP là như thế;
Mình chi tiết thêm 1 chút cho mọi người dễ nhìn
VP = $\frac12\left(\frac1{a^2+b^2+c^2}+\frac2{ab+bc+ca} \right)=\frac12\left(\frac1{a^2+b^2+c^2}+\frac{2^2 }{2(ab+bc+ca)}\right) \ge\frac12.\frac9{(a+b+c)^2} $
Phần còn lại thì dễ hiểu rồi.

Tiếp theo mời các bạn đến với bài 74

Bài 74 (Batigoal)
Cho các số thực $a,b,c >0 $ thỏa mãn $ab+bc+ca=1 $.Chứng minh rằng:

$\frac{a}{(3b+5c)^3}+\frac{b}{(3c+5a)^3}+\frac{c}{( 3a+5b)^3} \ge \frac{9}{512} $

[RIGHT][I][B]Nguồn: MathScope.ORG[/B][/I][/RIGHT]
 
__________________
“ Sức mạnh của tri thức là sự chia sẻ tri thức”

[Only registered and activated users can see links. ]
batigoal is offline   Trả Lời Với Trích Dẫn
Old 17-04-2011, 10:40 PM   #190
Kratos
+Thành Viên+
 
Tham gia ngày: Nov 2009
Đến từ: Toán 0912, PTNK, Tp.HCM
Bài gởi: 87
Thanks: 25
Thanked 160 Times in 73 Posts
Gửi tin nhắn qua Yahoo chát tới Kratos
Trích:
Nguyên văn bởi batigoal View Post
Bài 74 (Batigoal)
Cho các số thực $a,b,c >0 $ thỏa mãn $ab+bc+ca=1 $.Chứng minh rằng:

$\frac{a}{(3b+5c)^3}+\frac{b}{(3c+5a)^3}+\frac{c}{( 3a+5b)^3} \ge \frac{9}{512} $
Áp dụng Cauchy - Schwarz ta có

$LHS[a(3b+5c)] \ge \left (\dfrac{a}{3b+5c}+\dfrac{b}{3c+5a}+\dfrac{c}{3a+5b } \right)^2 $

Lại áp dụng Cauchy - Schwarz: $\dfrac{a}{3b+5c}+\dfrac{b}{3c+5a}+\dfrac{c}{3a+5b} \ge \dfrac{(a+b+c)^2}{8(ab+bc+ca)} $. Như vậy ta suy ra

$8(ab+bc+ca)LHS \ge \dfrac{(a+b+c)^4}{64(ab+bc+ca)^2}. $

Thay $ab+bc+ca $ bởi 1 với lưu ý $(a+b+c)^2 \ge 3(ab+bc+ca) $, ta suy ra đpcm. $\Box $
[RIGHT][I][B]Nguồn: MathScope.ORG[/B][/I][/RIGHT]
 

thay đổi nội dung bởi: Kratos, 17-04-2011 lúc 11:01 PM
Kratos is offline   Trả Lời Với Trích Dẫn
The Following 2 Users Say Thank You to Kratos For This Useful Post:
mrvui123 (16-09-2012), n.v.thanh (12-08-2011)
Old 17-04-2011, 10:44 PM   #191
Persian
+Thành Viên+
 
Tham gia ngày: Nov 2010
Đến từ: Có những thứ mình đã nhẫn tâm đánh mất sẽ không bao giờ lấy lại được.
Bài gởi: 257
Thanks: 103
Thanked 200 Times in 112 Posts
Trích:
Nguyên văn bởi batigoal View Post
Bài 74 (Batigoal)
Cho các số thực $a,b,c >0 $ thỏa mãn $ab+bc+ca=1 $.Chứng minh rằng:

$P=\frac{a}{(3b+5c)^3}+\frac{b}{(3c+5a)^3}+\frac{c} {(3a+5b)^3} \ge \frac{9}{512} $
Cauchy-schwarz kiểu này ra luôn
$\sum{a(3b+5c)}.P \geq (\sum{\frac{a}{3b+5c}})^2 $
mà $\sum{\frac{a}{3b+5c}} =\sum{\frac{a^2}{3ab+5c}}\geq \frac{3}{8} $và $\sum{a(3b+5c)} =8(ab+bc+ca)=8 $ nên dễ dàng có đpcm.

Đành post bài 75 vào đây -1 bài cũ.
Bài 75:Chứng minh rằng với mọi a;b;c ta có
$(a^2+2)(b^2+2)(c^2+2) \geq 3(a+b+c)^2 $
[RIGHT][I][B]Nguồn: MathScope.ORG[/B][/I][/RIGHT]
 
__________________

thay đổi nội dung bởi: Persian, 17-04-2011 lúc 10:48 PM
Persian is offline   Trả Lời Với Trích Dẫn
Old 18-04-2011, 11:05 AM   #192
daylight
+Thành Viên+
 
daylight's Avatar
 
Tham gia ngày: Dec 2009
Đến từ: Ha Noi
Bài gởi: 551
Thanks: 877
Thanked 325 Times in 188 Posts
Bài 75:Chứng minh rằng với mọi a;b;c ta có
$(a^2+2)(b^2+2)(c^2+2) \geq 3(a+b+c)^2 $

Giải: Đánh giá từ VP ta có

$(a+b+c)^2 \leq (a^2+2)[1+\frac{(b+c)^2}{2}] $

ta cần CM
$(b^2+2)(c^2+2) \geq 3[1+\frac{(b+c)^2}{2}] $
$\leftrightarrow \frac{b^2+c^2}{2}+b^2c^2-3bc+1 \geq 0 $
cái trên đúng vì $b^2+c^2 \geq 2bc $

Bài 76
Cho $a,b,c \geq 0 $
CMR
$\sqrt{\frac{a^3}{5a^2+(b+c)^2}}+\sqrt{\frac{b^3}{5 b^2+(a+c)^2}}+\sqrt{\frac{c^3}{5c^2+(a+b)^2}} \leq \sqrt{\frac{a+b+c}{3}} $
[RIGHT][I][B]Nguồn: MathScope.ORG[/B][/I][/RIGHT]
 
daylight is offline   Trả Lời Với Trích Dẫn
Old 18-04-2011, 11:50 AM   #193
batigoal
Super Moderator
 
batigoal's Avatar
 
Tham gia ngày: Jul 2010
Đến từ: Hà Nội
Bài gởi: 2,895
Thanks: 382
Thanked 2,968 Times in 1,295 Posts
Trích:
Nguyên văn bởi daylight View Post
[B][U]

Bài 76
Cho $a,b,c \geq 0 $
CMR
$\sqrt{\frac{a^3}{5a^2+(b+c)^2}}+\sqrt{\frac{b^3}{5 b^2+(a+c)^2}}+\sqrt{\frac{c^3}{5c^2+(a+b)^2}} \leq \sqrt{\frac{a+b+c}{3}} $
Em xem lại điều kiện của $a,b,c $ nhé Vì $a,b,c \geq 0 $ thì trong trường hợp $a=b=c=0? $

[RIGHT][I][B]Nguồn: MathScope.ORG[/B][/I][/RIGHT]
 
__________________
“ Sức mạnh của tri thức là sự chia sẻ tri thức”

[Only registered and activated users can see links. ]
batigoal is offline   Trả Lời Với Trích Dẫn
Old 18-04-2011, 07:32 PM   #194
Persian
+Thành Viên+
 
Tham gia ngày: Nov 2010
Đến từ: Có những thứ mình đã nhẫn tâm đánh mất sẽ không bao giờ lấy lại được.
Bài gởi: 257
Thanks: 103
Thanked 200 Times in 112 Posts
Trích:
Nguyên văn bởi daylight View Post
Bài 75:Chứng minh rằng với mọi a;b;c ta có
$(a^2+2)(b^2+2)(c^2+2) \geq 3(a+b+c)^2 $

Giải: Đánh giá từ VP ta có

$(a+b+c)^2 \leq (a^2+2)[1+\frac{(b+c)^2}{2}] $

ta cần CM
$(b^2+2)(c^2+2) \geq 3[1+\frac{(b+c)^2}{2}] $
$\leftrightarrow \frac{b^2+c^2}{2}+b^2c^2-3bc+1 \geq 0 $
cái trên đúng vì $b^2+c^2 \geq 2bc $

Bài 76
Cho $a,b,c \geq 0 $
CMR
$\sqrt{\frac{a^3}{5a^2+(b+c)^2}}+\sqrt{\frac{b^3}{5 b^2+(a+c)^2}}+\sqrt{\frac{c^3}{5c^2+(a+b)^2}} \leq \sqrt{\frac{a+b+c}{3}} $
$P = \sqrt {\frac{{{a^3}}}{{5{a^2} + {{(b + c)}^2}}}} + \sqrt {\frac{{{b^3}}}{{5{b^2} + {{(a + c)}^2}}}} + \sqrt {\frac{{{c^3}}}{{5{c^2} + {{(a + b)}^2}}}} \le \sqrt {\frac{{a + b + c}}{3}} \\
{P^2} \le (a + b + c)(\sum {\frac{{{a^2}}}{{5{a^2} + {{(b + c)}^2}}}} )\\ $
$5{a^2} + {(b + c)^2} = 2{a^2} + bc + 2{a^2} + bc + {a^2} + {b^2} + {c^2}\\ $
$\frac{{{a^2}({1^2} + {1^2} + {1^2})}}{{(2{a^2} + bc) + (2{a^2} + bc) + {a^2} + {b^2} + {c^2}}} \le \frac{{2{a^2}}}{{2{a^2} + bc}} + \frac{{{a^2}}}{{{a^2} + {b^2} + {c^2}}}\\ $
$\sum {\frac{{2{a^2}}}{{2{a^2} + bc}} + \frac{{{a^2}}}{{{a^2} + {b^2} + {c^2}}} \le 3} \\ $
$\sum {\frac{{{a^2}}}{{5{a^2} + {{(b + c)}^2}}}} \le \frac{1}{3} $
[RIGHT][I][B]Nguồn: MathScope.ORG[/B][/I][/RIGHT]
 
__________________
Persian is offline   Trả Lời Với Trích Dẫn
The Following 3 Users Say Thank You to Persian For This Useful Post:
daylight (23-04-2011), mrvui123 (16-09-2012), Thanh vien (21-04-2011)
Old 19-04-2011, 06:20 PM   #195
haptrung
+Thành Viên+
 
Tham gia ngày: Nov 2007
Bài gởi: 99
Thanks: 16
Thanked 31 Times in 23 Posts
Bài 77Cho a,b,c>0. Tìm giá trị nhỏ nhất của:
$\frac{2ab}{(c+a)(c+b)}+\frac{2bc}{(a+b)(a+c)} $+$\frac{3ca}{(b+c)(b+a)} $

[RIGHT][I][B]Nguồn: MathScope.ORG[/B][/I][/RIGHT]
 

thay đổi nội dung bởi: batigoal, 27-04-2011 lúc 05:49 AM Lý do: Đánh số
haptrung is offline   Trả Lời Với Trích Dẫn
Trả lời Gởi Ðề Tài Mới

Bookmarks

Ðiều Chỉnh
Xếp Bài

Quuyền Hạn Của Bạn
You may not post new threads
You may not post replies
You may not post attachments
You may not edit your posts

BB code is Mở
Smilies đang Mở
[IMG] đang Mở
HTML đang Tắt

Chuyển đến


Múi giờ GMT. Hiện tại là 01:19 PM.


Powered by: vBulletin Copyright ©2000-2024, Jelsoft Enterprises Ltd.
Inactive Reminders By mathscope.org
[page compression: 115.10 k/131.50 k (12.48%)]